Quanten.de Diskussionsforum

Quanten.de Diskussionsforum (http://www.quanten.de/forum/index.php5)
-   Quantenmechanik, Relativitätstheorie und der ganze Rest. (http://www.quanten.de/forum/forumdisplay.php5?f=3)
-   -   Lorentztransformation (http://www.quanten.de/forum/showthread.php5?t=577)

criptically 28.05.08 19:50

AW: Lorentztransformation
 
Zitat:

Zitat von rafiti (Beitrag 20341)
Kann ich nichts zu sagen, du provozierst aber schon, um zu solchen "Ergebnissen" zu kommen, um bspw. der Kritik etwas "wahres" im sinne von weiß-der-geier-was zu verleihen.

gruss
rafiti

Das ist falsch!
Ich habe nur gezeigt, dass die SRT rein mathematisch nicht gelten kann, nicht mehr und nicht weniger.
Jeder der etwas von Mathematik versteht, weiß dass aus
x'=x*sinα nur x=x'/sinα folgt.

Es kann nicht sein dass beides x'=x*sinα und x=x'*sinα parallel gilt. :D

mfg

criptically 28.05.08 19:52

AW: Lorentztransformation
 
Zitat:

Zitat von ingeniosus (Beitrag 20370)
Ja das ist eben räumlich 1-dimensional.

Jetzt die Lorenz-Transformation auf alle 3 Raumdimensionen anwenden und die SRT ist bewiesen.

Wenn man das noch die Gravitation dazunimmt hat man sogar Einsteins ART und RaumZeitKrümmung rechenbar verstanden.

Mehr ist es nicht.

Quatsch, da sind alle 4 Dimensionen (x,y,z,t) bzw. (x',y',z',t') .

mfg

rafiti 28.05.08 21:56

AW: Lorentztransformation
 
Zitat:

Zitat von criptically (Beitrag 20376)
Das ist falsch!
Ich habe nur gezeigt, dass die SRT rein mathematisch nicht gelten kann, nicht mehr und nicht weniger.
Jeder der etwas von Mathematik versteht, weiß dass aus
x'=x*sinα nur x=x'/sinα folgt.

Es kann nicht sein dass beides x'=x*sinα und x=x'*sinα parallel gilt. :D

mfg

Na klar, du weißt alles.

gruss
rafiti

Marco Polo 30.05.08 21:25

AW: Lorentztransformation
 
Zitat:

Zitat von Uli (Beitrag 20318)
Ich kann euch beiden hier überhaupt nicht folgen.
Was hat denn eine räumliche Drehung um einen Winkel alpha mit v oder c zu tun ?

Hi Uli,

das hast du scheinbar missverstanden. Die Rotation von Koordinatenachsen im Minkowski-Diagramm hat natürlich nichts mit einer räumlichen Drehung zu tun. Genausowenig wie dort schiefwinklige Koordinaten mit schief zueinander stehenden Koordinatenachsen im Raum zu tun haben.

Falls du es kapierst (ich kapier es nicht :o ), hier die mathematische Formulierung:

Die Gleichungen der Lorentztranformation beschreiben eine affine Abbildung, d.h. eine eineindeutige (bijektive), geradentreue und die Parallelität von Geraden erhaltende Abbildung der Ebene auf sich selbst.

In Minkowski-Diagrammen spielen v und damit auch ß die entscheidende Rolle schlechthin, wenn es um die Rotation von Koordinatenachsen geht.

Wir betrachten wie immer zwei Inertialsysteme S und S'.

Bei einer Relativgeschwindigkeit von v=(0,0,0) und t=t'=0 sind die ct-x-Achsen und die ct'-x'-Achsen natürlich deckungsgleich.

Wir gehen jetzt von einer Relativgeschwindigkeit v=(v,0,0) aus.

Wie drückt sich das im Minkowski-Diagramm aus?

Zitat:

tanphi=ß=v/c phi=arctanß phi=arctan(v/c)
Ganz einfach. Es wird nun die x'-Achse um phi=arctan(v/c) gegen den Uhrzeigersinn gedreht und die ct'-Achse um den gleichen Winkel mit dem Uhrzeigersinn gedreht.

Siehe hierzu auch

http://de.wikipedia.org/wiki/Lorentz...Minkowski-Raum

hier steht u.a.:

Zitat:

"Der Übergang von einem Bezugssystem auf ein relativ dazu bewegtes stellt sich im Minkowski-Raum dar als Übergang zu einem gedrehten, ebenfalls rechtwinkligen vierdimensionalen Bezugssystem. Der Drehwinkel φ hängt in der genannten Weise von der Relativgeschwindigkeit v der beiden Bezugssysteme ab:
tan φ = v/c "
Das sollte sich auch criptically mal durchlesen, damit er endlich mal von seinem sinus-trip herunter kommt.

Das Koordinatensystem des S'-System klappt sozusagen zusammen. Einen Notartzt muss man deswegen aber nicht rufen. :)

Ein Stab der Länge l, der im S-System im Koordinatenursprung ruht (also längs der x-Achse), hat dann im S'-System (also längs der um phi rotierten x'-Achse) eine andere Länge als im S-System (Längenkontraktion).

Zu beachten sind hier natürlich die unterschiedlichen Längeneinheiten auf den x bzw. x'-Achsen. Wenn wir auf der x-Achse die Längeneinheit L=1 haben, dann haben wir auf der um phi rotierten x'-Achse die Längeneinheit L=sqrt(gamma²+ß²gamma²) bzw. L=sqrt((1+ß²)/(1-ß²))

Mit etwas Übung (man muss jetzt z.B. zu der linken Weltlinie des Stabes eine ebenfalls schiewinklige parallele Weltlinie, also das rechte Ende, konstruieren usw.) kann man bequem die Länge des Stabes im S'-System ablesen.

1-2-3 ist keine Hexerei. :)

Grüssle,

Marco Polo

Marco Polo 30.05.08 21:38

AW: Lorentztransformation
 
Zitat:

Zitat von criptically (Beitrag 20377)
Quatsch, da sind alle 4 Dimensionen (x,y,z,t) bzw. (x',y',z',t') .

ingeniosus hat vollkommen Recht.

Die Lorentztransformation, so wie du sie korrekt angegeben hast, bezieht sich aber nur auf eine Bewegung in einer Raumdimension. Deswegen ist ja auch y'=y und z'=z.

Wenn wir aber eine Bewegung in 2 oder gar 3 Raumdimensionen betrachten, dann wird die Lorentztransformation wesentlich komplexer.

Wir haben dann nämlich nicht nur ß, sondern ßx, ßy und ßz zu berücksichtigen. Das kann man dann nur noch in Matrizenschreibweise darstellen.

Grüssle,

Marco Polo

criptically 30.05.08 22:01

AW: Lorentztransformation
 
Zitat:

Zitat von Marco Polo (Beitrag 20457)
ingeniosus hat vollkommen Recht.

Die Lorentztransformation, so wie du sie korrekt angegeben hast, bezieht sich aber nur auf eine Bewegung in einer Raumdimension. Deswegen ist ja auch y'=y und z'=z.

Wenn wir aber eine Bewegung in 2 oder gar 3 Raumdimensionen betrachten, dann wird die Lorentztransformation wesentlich komplexer.

Wir haben dann nämlich nicht nur ß, sondern ßx, ßy und ßz zu berücksichtigen. Das kann man dann nur noch in Matrizenschreibweise darstellen.

Grüssle,

Marco Polo

Wir können aber die Koordinaten immer so wählen, dass nur eine Koordinate betrachtet werden muss.

mfg

Marco Polo 30.05.08 22:10

AW: Lorentztransformation
 
Zitat:

Zitat von criptically (Beitrag 20317)
Ich bleibe dabei sinα=v/c .

Kannst du zeigen wie Koordinatensysteme gedreht sind, wenn man mit Tanges rechnet?

Nichts leichter als das.

Dazu betrachten wir einen Punkt im Minkowski-Diagramm.

Die Koordinaten:

(ct'=0 | x'=1)

Einsetzen in die Lorentzrücktransformation

ct=gamma(ct'+ßx') sowie x=gamma(x'+ßct') ergibt folgende Koordinaten

(ct=ß*gamma | x=gamma)

Jetzt konstruieren wir ein Steigungsdreieck mit den beiden Katheten gamma und ß*gamma.

tan(phi)=Gegenkathete/Ankathete

tan(phi)=ß*gamma/gamma

tan(phi)=ß

tan(phi)=v/c

Noch weitere Fragen?

Grüssle,

Marco Polo

criptically 30.05.08 22:17

AW: Lorentztransformation
 
Zitat:

Zitat von Marco Polo (Beitrag 20461)
Nichts leichter als das.

Dazu betrachten wir einen Punkt im Minkowski-Diagramm.

Die Koordinaten:

(ct'=0 | x'=1)

Einsetzen in die Lorentzrücktransformation

ct=gamma(ct'+ßx') sowie x=gamma(x'+ßct') ergibt folgende Koordinaten

(ct=ß*gamma | x=gamma)

Jetzt konstruieren wir ein Steigungsdreieck mit den beiden Katheten ß und ß*gamma.

tan(phi)=Gegenkathete/Ankathete

tan(phi)=ß*gamma/gamma

tan(phi)=ß

tan(phi)=v/c

Noch weitere Fragen?

Grüssle,

Marco Polo

Wie groß ist Winkel phi? :D

mfg

Marco Polo 30.05.08 22:20

AW: Lorentztransformation
 
Zitat:

Zitat von criptically (Beitrag 20458)
Wir können aber die Koordinaten immer so wählen, dass nur eine Koordinate betrachtet werden muss.

Angenommen wir haben zwei Inertialsysteme, die sich relativ zueinander mit v=(vx,vy,vz) bewegen.

Du hast zwar Recht, dass man die Koordinaten so wählen kann, dass nur eine der drei Geschwindigkeitskomponenten betrachtet werden muss.

Wenn das die Aufgabenstellung aber nicht vorsieht, was dann? Dann bekommst du in der Physik-Klausur eine glatte "sechs" und darfst dich setzen. :)

Grüssle,

Marco Polo

Marco Polo 30.05.08 22:22

AW: Lorentztransformation
 
Zitat:

Zitat von criptically (Beitrag 20462)
Wie groß ist Winkel phi? :D

phi=arctan(v/c) Ätsch!!! :D

criptically 30.05.08 22:31

AW: Lorentztransformation
 
Zitat:

Zitat von Marco Polo (Beitrag 20465)
phi=arctan(v/c) Ätsch!!! :D

Und der Winkel zwischen v und c? :D :D :D

mfg

Marco Polo 30.05.08 22:36

AW: Lorentztransformation
 
Zitat:

Zitat von criptically (Beitrag 20467)
Und der Winkel zwischen v und c? :D :D :D

Weitere Antworten nur noch gegen Bezahlung. :D

criptically 30.05.08 22:39

AW: Lorentztransformation
 
Zitat:

Zitat von Marco Polo (Beitrag 20468)
Weitere Fragen nur noch gegen Bezahlung. :D

Kann es sein dass v und c senkrecht aufeinander stehen? :D :D :D

Tangens=Gegenkathete/Ankathete !

mfg

Marco Polo 30.05.08 22:48

AW: Lorentztransformation
 
Zitat:

Zitat von criptically (Beitrag 20470)
Kann es sein dass v und c senkrecht aufeinander stehen? :D :D :D

Tangens=Gegenkathete/Ankathete !

Wer weiss? Kann schon sein. Korrekte Antwort erfolgt erst bei Überweisung von 20 Euro. :D

criptically 30.05.08 22:56

AW: Lorentztransformation
 
Zitat:

Zitat von Marco Polo (Beitrag 20473)
Wer weiss? Kann schon sein. Korrekte Antwort erfolgt erst bei Überweisung von 20 Euro. :D

Das sind Wucherpreise! :D :D

Zum Glück gibt es Wiki!

mfg

Marco Polo 31.05.08 07:42

AW: Lorentztransformation
 
Zitat:

Zitat von criptically (Beitrag 20376)
Das ist falsch!
Ich habe nur gezeigt, dass die SRT rein mathematisch nicht gelten kann, nicht mehr und nicht weniger.
Jeder der etwas von Mathematik versteht, weiß dass aus
x'=x*sinα nur x=x'/sinα folgt.

Es kann nicht sein dass beides x'=x*sinα und x=x'*sinα parallel gilt. :D

Von wegen. Du hast lediglich gezeigt, dass du keinen Schimmer hast.

Du hast das Prinzip leider immer noch nicht verstanden. Da helfen dir auch noch so viele breitgrinsende Smileys nicht weiter.

Es sollte auch dir allmählich klar werden, dass wenn sich zwei Inertialsysteme relativ zueinander bewegen, nur eines dasjenige sein kann, in dem ein bestimmter Vorgang ruht.

Damit ist immer eindeutig, welche der beiden Formeln zum Einsatz kommt.

Man darf beide Formeln daher nicht einfach gleichsetzen. Das wäre logisch völlig inkonsistent. Egal wie man es dreht. Bei der Betrachtung in einem Minkowski-Diagramm gilt immer nur eine der beiden Formeln und niemals beide zugleich, da der Vorgang den man betrachtet, immer nur in einem der beiden Inertialsysteme ruhen kann.

Wenn man die andere Formel anwenden möchte, dann muss der Vorgang den man betrachtet, eben im anderen Inertialsystem ruhen. Beides gleichzeitig geht halt nicht.

Beide Formeln gelten also, wie von dir behauptet, niemals parallel (auch nicht als Rücktrafo).

Das ist Kindergartenphysik, die du hier betreibst.

Abgesehen davon sind die Formeln, die du angegeben hast, auch noch falsch.

Leider hast du jetzt bereits mehrfach nachgewiesen, dass du enorme Wissenslücken bezüglich der SRT hast. Trotzdem posaunst du hier herum wie ein Elefant im Zirkus. Das sind doch eher mässige Voraussetzungen für einen SRT-Kritiker, gell?

Grüssle,

Marco Polo

JGC 31.05.08 08:39

AW: Lorentztransformation
 
Hallo Marco..


Ich wollte dir mal sagen, das es auch eine Frage der Betrachtung ist...

zu:

Zitat:

Es sollte auch dir allmählich klar werden, dass wenn sich zwei Inertialsysteme relativ zueinander bewegen, nur eines dasjenige sein kann, in dem ein bestimmter Vorgang ruht.
Das ist schon richtig...

Aber ist dir noch nie der Gedanke gekommen, das diese beiden jeweils zu betrachtenden Inertialsysteme zusammen eine übergeordnete Größe und Eigenschaft erhält??..

Beim Dreikörperproblem ist das doch schon offensichtlich..

Ein Objekt eines einzeln betrachteten Inertialsystems steht immer im Bezug mit einem anderen Objekt des jeweils anderen betrachteten Inertialsystems..

Das bedeutet doch letztlich, das beide Inertialsysteme in Wirklichkeit zusammen ein jeweils neues Inertialsystem bilden, deren Wichtigkeiten(Bedeutungsstärken in Form ihres jeweiligen "Machteinflusses") von Innen nach aussen hin abnehmen und nach einer jeweils vorgegebenen Reihenfolge wirksam werden

Einfach gesagt.. (aus drei Körper-Sicht A, B, C...)

Inertialsystem 1 = A > BC,

Inertialsystem 2 = B > AC,

Inertialsystem 3 = C > AB,

Das daraus übergeordnete Inertialsystem besteht dabei schon alleine auf Grund der Anwesenheit seiner zu ihm gehörigen 3 Körper ABC und kann als ein neues, "einziges" Initialsystem betrachtet werden, welches aus der Gesamtenergie und seiner gesamtsummierten Vektoren besteht...

Das also physikalisch gezeigte Eigenschaften im Grunde aus seinen "nicht gezeigten" (oder zumindest nicht offensichtlich gezeigten) elementaren Eigenschaften bestehen, welche die einzelnen Elemente einer Körpermenge von sich aus schon mitführen..

Die Elementareigenschaften einer Körpermenge bestimmen also doch im Grunde genommen die Eigenschaften der Menge als Ganzes(der "Einheitskörper")

Was sagst denn du dazu?


JGC

Marco Polo 31.05.08 09:03

AW: Lorentztransformation
 
Zitat:

Zitat von JGC (Beitrag 20483)
Das bedeutet doch letztlich, das beide Inertialsysteme in Wirklichkeit zusammen ein jeweils neues Inertialsystem bilden, deren Wichtigkeiten(Bedeutungsstärken in Form ihres jeweiligen "Machteinflusses") von Innen nach aussen hin abnehmen und nach einer jeweils vorgegebenen Reihenfolge wirksam werden

Halllo JGC,

bei Inertialsystemen gibt es aber nun mal keine Machteinflüsse, Wichtigkeiten oder Bedeutungsstärken. Alle sind gleichberechtigt.

Zitat:

Das daraus übergeordnete Inertialsystem besteht dabei schon alleine auf Grund der Anwesenheit seiner zu ihm gehörigen 3 Körper ABC und kann als ein neues, "einziges" Initialsystem betrachtet werden, welches aus der Gesamtenergie und seiner gesamtsummierten Vektoren besteht...
Es gibt auch keine übergeordneten Inertialsysteme. Ein Inertialsystem ist im Grunde doch nur eine spezielle Form eines Bezugssystems. Mehr nicht. Sozusagen ein Koordinatensystem, in dem der Trägheitssatz gilt.

Grüssle,

Marco Polo

JGC 31.05.08 14:05

AW: Lorentztransformation
 
Zitat:

Zitat von Marco Polo (Beitrag 20484)
Halllo JGC,

bei Inertialsystemen gibt es aber nun mal keine Machteinflüsse, Wichtigkeiten oder Bedeutungsstärken. Alle sind gleichberechtigt.



Es gibt auch keine übergeordneten Inertialsysteme. Ein Inertialsystem ist im Grunde doch nur eine spezielle Form eines Bezugssystems. Mehr nicht. Sozusagen ein Koordinatensystem, in dem der Trägheitssatz gilt.

Grüssle,

Marco Polo


Natürlich Marco...

Aber das gilt doch dann einfach immer nur für das entsprechende Koordinatensystem, welches man betrachten will.

Das ist also eine Entscheidung, die DU triffst, welches Koordinatensystem von dir bevorzugt wird... oder?

Was ist denn wenn mehrere Objekte jeweils nach ihrem Koordinatensystem wirken?


Jedes WAS bringt doch sein eigenes Koordinatensystem mit.(und wenn es noch so klein oder riesig ist)

Dein Beobachtungsstandort entscheidet doch, welches Koordinatensystem DU erst anwendest..

JGC

criptically 31.05.08 15:21

AW: Lorentztransformation
 
Zitat:

Zitat von Marco Polo (Beitrag 20482)
...

Es sollte auch dir allmählich klar werden, dass wenn sich zwei Inertialsysteme relativ zueinander bewegen, nur eines dasjenige sein kann, in dem ein bestimmter Vorgang ruht.

Damit ist immer eindeutig, welche der beiden Formeln zum Einsatz kommt.

Wieso sollte der Vorgang überhaupt ruhen? Was ist z.B. mit einem idealen Gas, wobei Milliarden und Abermilliarden von Molekülen in allen Richtungen durch die Gegend fliegen. Nur in einem Koordinatensystem befinden sie sich im statistischen Gleichgewicht.

Zitat:

Man darf beide Formeln daher nicht einfach gleichsetzen. Das wäre logisch völlig inkonsistent. Egal wie man es dreht. Bei der Betrachtung in einem Minkowski-Diagramm gilt immer nur eine der beiden Formeln und niemals beide zugleich, da der Vorgang den man betrachtet, immer nur in einem der beiden Inertialsysteme ruhen kann.
Wieso denn nicht? Was ist z.B. mit Zwillingsparadoxon, wobei beide Zwillinge wegen symmetrischer Betrachtungsweise gleich alt sein müssen?

Zitat:

Wenn man die andere Formel anwenden möchte, dann muss der Vorgang den man betrachtet, eben im anderen Inertialsystem ruhen. Beides gleichzeitig geht halt nicht.

Beide Formeln gelten also, wie von dir behauptet, niemals parallel (auch nicht als Rücktrafo).

Das ist Kindergartenphysik, die du hier betreibst.

Abgesehen davon sind die Formeln, die du angegeben hast, auch noch falsch.

Leider hast du jetzt bereits mehrfach nachgewiesen, dass du enorme Wissenslücken bezüglich der SRT hast. Trotzdem posaunst du hier herum wie ein Elefant im Zirkus. Das sind doch eher mässige Voraussetzungen für einen SRT-Kritiker, gell?

Grüssle,

Marco Polo
Die Formeln habe ich aus einem Standardbuch übernommen, warum sollen die falsch sein, wenn sie dort drin stehen? :D :D

mfg

pauli 31.05.08 15:27

AW: Lorentztransformation
 
Zitat:

Zitat von criptically (Beitrag 20496)
Wieso denn nicht? Was ist z.B. mit Zwillingsparadoxon, wobei beide Zwillinge wegen symmetrischer Betrachtungsweise gleich alt sein müssen?

Genau, schliesslich beschleunigt einer von ihnen mehrmals, also ist die Betrachtungsweise symmetrisch und die RT zum 8. mal heute widerlegt!

Uranor 31.05.08 16:07

AW: Lorentztransformation
 
Zitat:

Zitat von criptically (Beitrag 20496)
Wieso sollte der Vorgang überhaupt ruhen? Was ist z.B. mit einem idealen Gas, wobei Milliarden und Abermilliarden von Molekülen in allen Richtungen durch die Gegend fliegen. Nur in einem Koordinatensystem befinden sie sich im statistischen Gleichgewicht.

Was hätten die Molleküle gemeinsam? Jedes ruht im eigenen Inertialsystem. Relativität lässt sich prima an einem idealen Gas zeigen. Unser Hausmaler soll mal jedem der Molleküle eine individuelle Farbe verpassen.


Zitat:

Zitat von criptically (Beitrag 20496)
Wieso denn nicht? Was ist z.B. mit Zwillingsparadoxon, wobei beide Zwillinge wegen symmetrischer Betrachtungsweise gleich alt sein müssen?

Wenn die beiden symmetrisch betrachtet und damei als gleich alt bewertet werden müssen, sind sie gleich alt. Z.B. kann der eine Zwilling relativ schneller sein, der andere kann sich dazu passend tiefer im G-Feld aufhalten. Der Zusammenhang wäre eindeutig und unabhängig von beliebigen Betrachtungsorten.

Berechnung oder Uhrenvergleich. A priori wird man den Zusammenhang nicht erkennen können.


Gruß Uranor

criptically 31.05.08 16:20

AW: Lorentztransformation
 
Zitat:

Zitat von pauli (Beitrag 20498)
Genau, schliesslich beschleunigt einer von ihnen mehrmals, also ist die Betrachtungsweise symmetrisch und die RT zum 8. mal heute widerlegt!

Falsch, es geht um 2 Zwillinge, welche im Bezug zur Erde in entgegengesetzten Richtungen beschleunigt werden. :D :D

Relativ untereinander haben sie jeweils gleiche aber entgegengesetzte Geschwindigkeiten. Relativ zu einem Erdbewohner altern sie gleich schnell, aber relativ zueinander altern sie jeweils langsamer. :D :D :D

Das wäre die 9. Widerlegung heute.

mfg

criptically 31.05.08 16:23

AW: Lorentztransformation
 
Zitat:

Zitat von Uranor (Beitrag 20499)
...

Berechnung oder Uhrenvergleich. A priori wird man den Zusammenhang nicht erkennen können.


Gruß Uranor

Und wenn sie unterwegs die Uhren untereinander tauschen? Haben sie auch die Zeit getauscht? :D :D :D

mfg

Uranor 31.05.08 16:59

AW: Lorentztransformation
 
Zitat:

Zitat von criptically (Beitrag 20502)
Und wenn sie unterwegs die Uhren untereinander tauschen? Haben sie auch die Zeit getauscht? :D :D :D

mfg

Ach wo. Sie haben für den Vorgang Zeit verbraucht. ;)


mfg

criptically 31.05.08 22:03

AW: Lorentztransformation
 
Zitat:

Zitat von Uranor (Beitrag 20506)
Ach wo. Sie haben für den Vorgang Zeit verbraucht. ;)


mfg

Also die Zwillinge sollten mit der Zeit sparsamer umgehen. :D
Mit welcher Geschwindigkeit läuft meine Zeit zu dir und mit welcher läuft deine Zeit zu mir? :D :D
Kann ich deine Zeit mit meiner Uhr irgendwie einfangen? :D :D

mfg

Uranor 31.05.08 22:27

AW: Lorentztransformation
 
Zitat:

Zitat von criptically (Beitrag 20517)
Also die Zwillinge sollten mit der Zeit sparsamer umgehen. :D
Mit welcher Geschwindigkeit läuft meine Zeit zu dir und mit welcher läuft deine Zeit zu mir? :D
Kann ich deine Zeit mit meiner Uhr irgendwie einfangen? :D :D

mfg

Geh, lass Zeit zumindetz in der kommenden Woche nicht laufen. Was sollen denn die Zaungäste denken? :p :p

mfg

Uli 01.06.08 12:25

AW: Lorentztransformation
 
Hi Marco Polo,

ich finde diesen Wikipedia-Artikel allerdings ziemlich "schwierig" - v.a. im Abschnitt
"Warum verschwindet bei der Drehung die X'-Achse nicht aus unserem Erfahrungsraum?"
verstehe ich nur noch "Bahnhof". Ist das nicht wirklich konfus oder bin ich nur so blöd ?

Gruß,
Uli

Marco Polo 01.06.08 17:37

AW: Lorentztransformation
 
Zitat:

Zitat von Uli (Beitrag 20559)
ich finde diesen Wikipedia-Artikel allerdings ziemlich "schwierig" - v.a. im Abschnitt
"Warum verschwindet bei der Drehung die X'-Achse nicht aus unserem Erfahrungsraum?"
verstehe ich nur noch "Bahnhof". Ist das nicht wirklich konfus oder bin ich nur so blöd ?

Ja Uli,

den Abschnitt finde ich auch etwas eigenartig. "Die verschiedenen Höhenlagen der vierten Dimension". Hä?

Oder dass die verschiedenen x'-Achsen (deren Summe eine Ebene darstellt), alle gleichzeitig anwesend sein sollen, finde ich auch komisch.

O.K.. Alle diese x'-Achsen sind zwar möglich, werden aber imho erst bei einer konkreten Situation verwirklicht. Verwirklicht in dem Sinne, dass man dann anhand der zugehörigen Relativgeschwindigeit Berechnungen zwischen Inertialsystemen vornehmen kann.

Jetzt könnte man sich natürlich unendlich viele relativ zu sich bewegte Inertialsysteme vorstellen. Dann kommt das mit der Ebene schon hin. Und dann sind auch alle x'-Achsen vorhanden.

Diese müssten dann aber alle durch den 0-Punkt gehen und nicht parallel zur x'-Achse laufen. Die Parallelen, die dort eingezeichnet sind, sind lediglich Linien der Gleichzeitigkeit. Alle Ereignisse, die im Monkowski-Diagramm auf einer Geraden parallel zur x-Achse oder x'-Achse liegen, geschehen aus Sicht des jeweiligen Inertialsystems S oder S' gleichzeitig. Na ja, das ist ja auch logisch.

Die Formulierung im Wiki-Artikel, ist daher imho irreführend.

Grüssle,

Marco Polo

Marco Polo 01.06.08 21:23

AW: Lorentztransformation
 
Zitat:

Zitat von JGC (Beitrag 20493)
Aber das gilt doch dann einfach immer nur für das entsprechende Koordinatensystem, welches man betrachten will.

Das ist also eine Entscheidung, die DU triffst, welches Koordinatensystem von dir bevorzugt wird... oder?

Was ist denn wenn mehrere Objekte jeweils nach ihrem Koordinatensystem wirken?


Jedes WAS bringt doch sein eigenes Koordinatensystem mit.(und wenn es noch so klein oder riesig ist)

Dein Beobachtungsstandort entscheidet doch, welches Koordinatensystem DU erst anwendest..

Hi JGC,

nochmal zum mitschreiben. Es gibt keine bevorzugten Bezugssysteme. Nur weil ich mich gedanklich in das eine oder andere Bezugssystem begebe und von diesem aus meine Berechnungen durchführe, bedeutet das doch nicht, dass ich dieses oder jenes Bezugssystem in jedweder Art bevorzuge.

Wenn ich mich für ein Bezugssystem entscheide aus dem heraus ich meine Berechnungen durchführen möchte, dann geschieht dies doch wohl eher aus wissenschaftlicher Neugier und nicht weil ich dieses Bezugssystem in irgendeiner Weise bevorzuge. Hierzu solllte man noch wissen, dass nicht jedes Bezugssystem ein Inertialsystem ist. Jedes Inertialsystem ist allerdings ein Bezugssystem, allerdings ein spezielles.

Es gibt auch keine kleinen oder riesigen Koordinatensysteme. Ein Koordinatensystem ist ein Koordinatensystem. Die Dinger sind nach allen Seiten hin unbegrenzt.

Grüssle,

Marco Polo

Marco Polo 01.06.08 21:40

AW: Lorentztransformation
 
Zitat:

Zitat von criptically (Beitrag 20317)
Ich bleibe dabei sinα=v/c .

Kannst du zeigen wie Koordinatensysteme gedreht sind, wenn man mit Tanges rechnet?

So, ich hatte ja unlängst hergeleitet, dass der Drehwinkel im Minkowski-Diagramm phi=arctan(v/c) ist.

Jetzt sei bitte mal so nett und leite her, dass sinα=v/c sei.

Darauf bin ich wirklich mal gespannt. Und bitte keine Ablenkungen. Einfach mal eine vernünftige Herleitung ohne deine üblichen Gegenfragen mit Smileys versehen.

Ich wette, das bekommst du nicht hin.

Grüssle,

Marco Polo

criptically 01.06.08 21:55

AW: Lorentztransformation
 
Zitat:

Zitat von Marco Polo (Beitrag 20594)
So, ich hatte ja unlängst hergeleitet, dass der Drehwinkel im Minkowski-Diagramm phi=arctan(v/c) ist.

Jetzt sei bitte mal so nett und leite her, dass sinα=v/c sei.

Darauf bin ich wirklich mal gespannt. Und bitte keine Ablenkungen. Einfach mal eine vernünftige Herleitung ohne deine üblichen Gegenfragen mit Smileys versehen.

Ich wette, das bekommst du nicht hin.

Grüssle,

Marco Polo

Wozu brauche ich Minkowski? :D

sin²α+cos²α=1 ;

cos²α=1-sin²α ;

cosα=√(1-sin²α) ;

mit sinα=v/c ->cosα=√(1-v²/c²) ;

Anwendung Längenkontraktion

L'=L*cosα=L*√(1-v²/c²) ! :D :D

mfg

Marco Polo 01.06.08 22:24

AW: Lorentztransformation
 
Zitat:

Zitat von criptically (Beitrag 20597)
Wozu brauche ich Minkowski? :D

sin²α+cos²α=1 ;

cos²α=1-sin²α ;

cosα=√(1-sin²α) ;

mit sinα=v/c ->cosα=√(1-v²/c²) ;

Anwendung Längenkontraktion

L'=L*cosα=L*√(1-v²/c²) ! :D :D

mfg

Dass sin²α+cos²α=1 ist, weiss ich auch. Und dass daraus folgt, dass cos²α=1-sin²α ist und damit für cosα=√(1-sin²α) gilt, ist unschwer nachvollziehbar, auch wenn für cos²α=0,5*(1+cos2α) gängiger sein dürfte.

Was hat das jetzt mit dem Drehwinkel im Minkowski-Diagramm zu tun?

Auch wenn du Minkowski nicht zu brauchen scheinst, bleibst du den Nachweis, dass sinα=v/c der Drehwinkel dort sei, immer noch schuldig.

Was soll also deine spinnerte Formel cosα=√(1-v²/c²)? Ich kann aus den Winkelfunktionen die unglaublichsten Formeln konstruieren.

Also was ist denn jetzt bitte mit deinem Beweis? Schliesslich hattest du behauptet

Zitat:

Ich bleibe dabei sinα=v/c .

Kannst du zeigen wie Koordinatensysteme gedreht sind, wenn man mit Tanges rechnet?
Da ich das kürzlich gezeigt habe, kannst du es ja jetzt mit dem Sinus zeigen. Das steht immer noch aus...

Bitte mit Angabe von Koordinaten im Minkowski-Diagramm.

JGC 01.06.08 23:32

AW: Lorentztransformation
 
Zitat:

Zitat von Marco Polo (Beitrag 20592)
Hi JGC,

nochmal zum mitschreiben. Es gibt keine bevorzugten Bezugssysteme. Nur weil ich mich gedanklich in das eine oder andere Bezugssystem begebe und von diesem aus meine Berechnungen durchführe, bedeutet das doch nicht, dass ich dieses oder jenes Bezugssystem in jedweder Art bevorzuge.

Wenn ich mich für ein Bezugssystem entscheide aus dem heraus ich meine Berechnungen durchführen möchte, dann geschieht dies doch wohl eher aus wissenschaftlicher Neugier und nicht weil ich dieses Bezugssystem in irgendeiner Weise bevorzuge. Hierzu solllte man noch wissen, dass nicht jedes Bezugssystem ein Inertialsystem ist. Jedes Inertialsystem ist allerdings ein Bezugssystem, allerdings ein spezielles.

Es gibt auch keine kleinen oder riesigen Koordinatensysteme. Ein Koordinatensystem ist ein Koordinatensystem. Die Dinger sind nach allen Seiten hin unbegrenzt.

Grüssle,

Marco Polo

Du entscheidest aber und legst somit deren jeweiligen Gültigkeits-Prioritäten fest...

Marco Polo 01.06.08 23:39

AW: Lorentztransformation
 
Zitat:

Zitat von JGC (Beitrag 20602)
Du entscheidest aber und legst somit deren jeweiligen Gültigkeits-Prioritäten fest...

Papperlapapp. Schliesslich kann ich mich auch dafür entscheiden, alle Bezugssysteme gleichzeitig zu berücksichtigen und alle parallel zu betrachten.

Aber selbst wenn ich mich jetzt für eines entscheide, ist dieses nicht bevorzugt. Und wenn, dann eben nur aus meiner beschränkten Sicht.

Abgesehen davon brauche ich mich ja gar nicht entscheiden. Ich weiss auch so, dass alle Bezugssysteme gleichberechtigt sind.

criptically 02.06.08 21:12

AW: Lorentztransformation
 
Zitat:

Zitat von Marco Polo (Beitrag 20600)
Dass sin²α+cos²α=1 ist, weiss ich auch. Und dass daraus folgt, dass cos²α=1-sin²α ist und damit für cosα=√(1-sin²α) gilt, ist unschwer nachvollziehbar, auch wenn für cos²α=0,5*(1+cos2α) gängiger sein dürfte.

Was hat das jetzt mit dem Drehwinkel im Minkowski-Diagramm zu tun?

Was geht mich Minkowski an. Meine Gleichung ist eine Erfahrungstatsache und die ist mit Physik völlig konform. Daraus ergibt sich leider Folgerung dass die RT nur eine Transformation von wahren Größen auf Scheingrößen ist. So z.B. ist cosα=c'/c, wobei c' die LG in einem senkrecht zur Ausbreutungsrichtung bewegten Koordinatensystem ist. Also c'=c*cosα=c*√(1-v²/c²)

Zitat:

Auch wenn du Minkowski nicht zu brauchen scheinst, bleibst du den Nachweis, dass sinα=v/c der Drehwinkel dort sei, immer noch schuldig.

Was soll also deine spinnerte Formel cosα=√(1-v²/c²)? Ich kann aus den Winkelfunktionen die unglaublichsten Formeln konstruieren.

Also was ist denn jetzt bitte mit deinem Beweis? Schliesslich hattest du behauptet



Da ich das kürzlich gezeigt habe, kannst du es ja jetzt mit dem Sinus zeigen. Das steht immer noch aus...

Bitte mit Angabe von Koordinaten im Minkowski-Diagramm.
Du musst mehr lesen.

Zitat:

Für die grafische Übersetzung der Koordinaten muss jedoch berücksichtigt werden, dass in diesem Diagramm die Maßstäbe auf den gekippten Achsen länger sind als im oben geschilderten newtonschen Fall. Um dieses Problem zu umgehen, empfiehlt sich eine Verformung des gesamten Diagramms derart, dass der Maßstab auf sämtlichen Achsen gleich wird, während die Zuordnung von x und t zu x' und t' unverändert bleibt. Das gelingt mit einer Stauchung in Richtung 45° oder auch einer Streckung in Richtung 135° bis zu der Situation, in der die Winkelhalbierende der Zeitachsen auf der der Wegachsen senkrecht steht. Für den Winkel β zwischen den beiden Zeit- beziehungsweise Wegachsen gilt

\sin(\beta) = \frac{v}{c} .

In dieser symmetrischen Darstellung sind beide Beobachter beziehungsweise Koordinatensysteme völlig gleichwertig dargestellt.

Minkowski-Diagramm in der speziellen Relativitätstheorie
mfg

JGC 02.06.08 21:39

AW: Lorentztransformation
 
Zitat:

Zitat von Marco Polo (Beitrag 20603)
Papperlapapp. Schliesslich kann ich mich auch dafür entscheiden, alle Bezugssysteme gleichzeitig zu berücksichtigen und alle parallel zu betrachten.

Aber selbst wenn ich mich jetzt für eines entscheide, ist dieses nicht bevorzugt. Und wenn, dann eben nur aus meiner beschränkten Sicht.

Abgesehen davon brauche ich mich ja gar nicht entscheiden. Ich weiss auch so, dass alle Bezugssysteme gleichberechtigt sind.

Hi...

Woher weißt du, wie sich die in ihren jeweilig gewählten/betrachteten Bezugssystemen stattfindenden Ereignisse jeweils zueinander verhalten?

Ein Ereignis, das jetzt von links auf dich mit LG zukommt, wirkt auch von links, erscheint dir aber anders, wie einem Beobachter, der dieses Ereignis frontal erfassen/erleben kann. Seitliche Wellen kannst du ja schliesslich nicht sehen.. Nur ihre Wirkungen beobachten.

Hast du denn schon mal einen Lichtstrahl "von der Seite " gesehen?

Du kannst ihn aber zumindest in Form von einer EM-Wirkung messen

Ich will es mal ausführlicher darlegen..

Stell dir vor, du stehst in deinem Bezugssystem(dein "Jetzt und hier") und du befindest dich quasi mitten im Universum..

Jetzt bewegen sich all die darin beobachtbaren Ereignisse relativ zu dir.. (nach oben/unten, Nach links/rechts und zusätzlich noch von dir weg, oder auf dich zu...)

Ein Lichtstrahl, der senkrecht auf dein Auge in die Netzhaut fällt(also senkrecht zu deiner Sichtrichtung...)kommt immer direkt mit LG vom fassbaren Objekt zu dir...

Das heißt, das all die Lichtstrahlen, die direkt auf deine Augen/Sensoren treffen, ihr "wahres Gesicht" in Form einer elektromagnetischen Abbildung eines Bildes in deine geistige Grafikkarte des Gehirns auf den Monitor deiner Wahrnehmung pinselt. Eine "flache"(zeitlose) Momentaufnahme...

Und nicht vergessen... Bei LG herrscht Zeitlosigkeit!

Was aber mit all den anderen Lichtstrahlen, die nicht genau im 90°Winkel auf deine Netzhaut oder der Messfläche prallen?

Sie treffen phasenverschoben(durch den, je nach auftretender Winkelveränderungen veränderten/Brechungsindex) auf die Sensoroberfläche auf und werden dadurch in andere Spektren gebeugt(transformiert)als ursprünglich im sichtbaren Licht gezeigt, da sie ja insgesamt ihre jeweiligen "Strahl-Geschwindigkeiten" beibehalten müssen(konstante LG)..

Du verstehst?

Meiner Meinung nach erfährt dabei ein Signal eine scheinbare Laufzeitveränderung, die sich durch Winkel-Abweichungen der senkrechten Blick-Ebene in einer veränderten Wahrnehmungsweise äussert, weil ja das Messgerät oder der Interpreter ja ebenso auf seine Licht-Geschwindigkeit "geeicht" ist...

Und so wird dann das tatsächliche Geschehen unserer Wahrnehmung gegenüber "verborgen"


Ich hab mal aus Gaudi so ein Bild aus dem Spektrum-Verlag genommen, um zu zeigen, wie ein Bild des Kosmos erscheint, wenn ich ihm alle EM-Wahrnehmungen gleichzeitig übereinander lege(Also im sichtbaren und messbaren Bereich..

Die Gravitation hab ich als Helligkeit in Tiefe verwandelt, um den Effekt besser sichtbar zu machen

Herausgekommen ist dabei dieses..

http://www.clausschekonstanten.de/schau/rel_1.jpg

http://www.clausschekonstanten.de/schau/rel_2.jpg

http://www.clausschekonstanten.de/schau/rel_3.jpg

http://www.clausschekonstanten.de/schau/rel_4.jpg
Wie eine Sat-Aufnahme eines belebten Planetens...


So könnte das Universum in etwa aussehen, wenn wir in alle Koordinaten-Achsrichtungen gleichzeitig bei LG existieren würden...

So sind zumindest bestimmte Sichtweisen von mir, da doch Photonen selbst kein Alter kennen, und es sich uns gegenüber doch irgendwie bemerkbar machen muss, wenn wir nicht mit LG im Raume existieren...


JGC

rafiti 02.06.08 21:53

AW: Lorentztransformation
 
Zitat:

Zitat von criptically (Beitrag 20611)
Was geht mich Minkowski an. Meine Gleichung ist eine Erfahrungstatsache und die ist mit Physik völlig konform. Daraus ergibt sich leider Folgerung dass die RT nur eine Transformation von wahren Größen auf Scheingrößen ist. So z.B. ist cosα=c'/c, wobei c' die LG in einem senkrecht zur Ausbreutungsrichtung bewegten Koordinatensystem ist. Also c'=c*cosα=c*√(1-v²/c²)



Du musst mehr lesen.



mfg

Das ist wieder so ein Rumgelalle von dir, was soll eine Scheingröße denn sein, erklär mal den Unterschied?

gruss
rafiti

Uranor 02.06.08 22:21

AW: Lorentztransformation
 
salve JGC,

bekämest du diverse Einblicke auch kachelbar hin? Bzw. als Hintergrund(-geschehen) für ein dynamisches Szenario im Vordergrund? Bzw. die Operater und Gerätschaften des Szenarios im Vordergrund? Dein Talent würde prima nutz- und einsetzbar sein.

Gruß Uranor

JGC 03.06.08 06:46

AW: Lorentztransformation
 
Zitat:

Zitat von Uranor (Beitrag 20616)
salve JGC,

bekämest du diverse Einblicke auch kachelbar hin? Bzw. als Hintergrund(-geschehen) für ein dynamisches Szenario im Vordergrund? Bzw. die Operater und Gerätschaften des Szenarios im Vordergrund? Dein Talent würde prima nutz- und einsetzbar sein.

Gruß Uranor

Äh..

Wie "Kachelbar"

Ich mein, mach dir die Bilder runter, verkleinere sie auf gewünschte Grösse(oder entsprechenden Ausschnitt) speichere sie als BMP-Bitmap ab und benutze sie im Desktop Hintergrund mit Option gekachelt..

Oder wolltest du sowas eher als Animation sehen?(wie ein brodeln)

Das wäre schon schwieriger, es gibt aber Programme, die das scheinbar können, gesehen hab ich mal sowas..

Schönen Sonntag noch..

Uranor 03.06.08 10:33

AW: Lorentztransformation
 
salve JGC,

"Kachelbar" meint, dass man sie wie Kacheln setzen kann, aber keine Ünergänge sieht. Die Bilder sind dann so aufgebaut, dass sie ohne Motivbruch ineinander übergehen.

Äh nein, nicht für den Desktop. Sonnen, Planeten, dein 100 m Hyperraumschiff wären im Vordergrund. Die gekackelten Bilder würden den Hintergrund ergeben. Eine andere Anwendung für dezenteren Aufbau wäre der Hintergrund von Webseiten.

Jau, die Nutzung wäre projektseitig. Deine Effekte können ja für verschiedene Zwecke speziell entworfen werden. Und jetzt, wo ich gesehen hab, dass du dich durchaus auch schon mal einem Raumer widmest... na ja, positiv genutztes Talent eben. Man glaubt als Nichtadept gar nicht, wie viel 2D-Grafik benötigt wird. Denk nur an Texturen aller Art. Aber es war halt nur mal eine Idee, da du dein Talent so gern einsetzt.

Gruß Uranor

JGC 03.06.08 21:42

AW: Lorentztransformation
 
Ah, jetzt hab ich verstanden....

Kachelbar heißt, man teilt ein Bild mittels Mittelkreuz in 4 Teile und dreht deren Innenseiten nach außen und retuschiert das "Spiegel-Kreuz"..

Ich probier es mal, nur jetzt grade nicht...


Keine Bange, den Flieger scannte ich aus einem Fan-Heft und stellte ihn frei..

Alles selbst erstellen dauert viel Zeit und ich bin froh, wenn ich nicht den ganzen Tag vor der Kiste hock... Dann braucht´s bei mir eben auch eine konkrete Vorstellung, WIE ich WAS realisieren kann, WAS mir dazu einfällt, meine Ruhe, Lust und Motivation, sonst glotz ich nur ein Loch in den Schirm und seh nix...


JGC

Uranor 03.06.08 23:11

AW: Lorentztransformation
 
Jau. Wichtig ist, dass sich das Bild an den Kantenübergängen bruchfrei fortsetzt. Charakter und Größe der Muster richten sich individuell nach dem Bedarf. Tapete, große Bodenkacheln, "die unendlichen Weiten" als Hintergrund für z.B. das Kopflogo-Bild einer HP.

Das, was du ansprichst, kenne ich in meinem Fall halt textseitig. Mitunter verlangt der Körper nach völlig anderen Perspektiven, ein andermal will die Arbeit flutschen. Ob sie dann was taugte, will dann gecheckt werden. Das ganze ist halt langzeitlich ausgelegt. Es ist üblich, dass am Anfang erst mal die meisten Spezialisten fehlen. Appetit kommt beim Futtern. Also soll der Startbereich möglichst ansprechend gelingen. Geduld und Muße, oder besser gar nicht erst beginnen...


Ach so. Natürlich muss etwas, dass man öffentlich präsentiert, auf eigener Arbeit beruhen. Es geht um Copyrhigt, aber auch sehr um den Ehrgeiz. Anregungen holen ist oh ja ganz wichtig. Aber Umsetzungen incl. Ideengut wollen unbedingt die eigene Handschrift kosten.

Gruß Uranor

ingeniosus 05.06.08 17:31

AW: Lorentztransformation
 
Zitat:

Zitat von Marco Polo (Beitrag 20592)
Hi JGC,

nochmal zum mitschreiben. Es gibt keine bevorzugten Bezugssysteme. Nur weil ich mich gedanklich in das eine oder andere Bezugssystem begebe und von diesem aus meine Berechnungen durchführe, bedeutet das doch nicht, dass ich dieses oder jenes Bezugssystem in jedweder Art bevorzuge.

Du bevorzugst es ja schon, indem Du es auswählst, um alle Bezugssysteme darauf zu beziehen.

Aber Du beeinflusst durch die Wahl dieses Bezugssystemes nicht die Abläufe der einzelnen Systeme untereinander.

Du fixierst sozusagen nur Deinen Beobachtungspunkt!

criptically 05.06.08 17:57

AW: Lorentztransformation
 
Zitat:

Zitat von rafiti (Beitrag 20615)
Das ist wieder so ein Rumgelalle von dir, was soll eine Scheingröße denn sein, erklär mal den Unterschied?

gruss
rafiti

Auf dem Bild unten ist eine wahre Größe (Kraft) in grün und Scheingröße in rot dargestellt.

http://www.phynet.de/upload/Mechanik...oraddition.jpg

mfg

ingeniosus 06.06.08 18:57

AW: Lorentztransformation
 
Zitat:

Zitat von criptically (Beitrag 20611)
Was geht mich Minkowski an. Meine Gleichung ist eine Erfahrungstatsache und die ist mit Physik völlig konform. Daraus ergibt sich leider Folgerung dass die RT nur eine Transformation von wahren Größen auf Scheingrößen ist. So z.B. ist cosα=c'/c, wobei c' die LG in einem senkrecht zur Ausbreutungsrichtung bewegten Koordinatensystem ist. Also c'=c*cosα=c*√(1-v²/c²)

Minkowski ist aber schon ein bekannter Theoretiker.

Die RT transformiert wahre Grössen in echte Rechengrössen, aber in keine Scheingrössen!

Die Formel wäre eine Standard-Ablenkung, was ist daran bemerkenswert?

Marco Polo 06.06.08 21:29

AW: Lorentztransformation
 
Zitat:

Zitat von criptically (Beitrag 20611)
Was geht mich Minkowski an. Meine Gleichung ist eine Erfahrungstatsache und die ist mit Physik völlig konform. Daraus ergibt sich leider Folgerung dass die RT nur eine Transformation von wahren Größen auf Scheingrößen ist. So z.B. ist cosα=c'/c, wobei c' die LG in einem senkrecht zur Ausbreutungsrichtung bewegten Koordinatensystem ist. Also c'=c*cosα=c*√(1-v²/c²)

Was geht dich Minkowski an? Du hast den doch ins Spiel gebracht mit deinem Sinus-Quatsch. Davon mal abgesehen. Was gehen uns deine Gleichungen an, die angeblich Erfahrungstatsachen sind?

c'=c*cosα=c*√(1-v²/c²)

Hihi. Oh mann. Was für ein Unsinn.

Hattest du nicht gefragt, wie man den Drehwinkel mit dem Tangens darstellen kann? Ich hatte das dann erklärt, und dann kommt dein Einwand, was geht mich Minkowski an? Sag mal, gehts noch?

Marco Polo 06.06.08 21:31

AW: Lorentztransformation
 
Zitat:

Zitat von criptically (Beitrag 20697)
Auf dem Bild unten ist eine wahre Größe (Kraft) in grün und Scheingröße in rot dargestellt.

http://www.phynet.de/upload/Mechanik...oraddition.jpg

Das ist keine Scheinkraft, sondern eine resultierende Kraft.

Marco Polo 06.06.08 21:37

AW: Lorentztransformation
 
Zitat:

Zitat von ingeniosus (Beitrag 20696)
Du bevorzugst es ja schon, indem Du es auswählst, um alle Bezugssysteme darauf zu beziehen.

Nein. Da wird gar nichts bevorzugt. Das ist doch Unsinn. Eine Bevorzugung bedeutet in der Physik, dass es sich um ein ausgezeichnetes Bezugssystem handelt. So etwas existiert überhaupt nicht.

Bevorzugung höchstens in dem Sinne, dass ich mich für eines entschieden habe. Dadurch zeichnet sich das ausgewählte Bezugssystem aber nicht vor den anderen aus. Das ist das 1. Einstein´sche Postulat.

Marco Polo 06.06.08 23:48

AW: Lorentztransformation
 
Zitat:

Zitat von criptically (Beitrag 20496)
Wieso denn nicht? Was ist z.B. mit Zwillingsparadoxon, wobei beide Zwillinge wegen symmetrischer Betrachtungsweise gleich alt sein müssen?

Ohne die Beschleunigungsphasen ist alles symmetrisch. Leider kommt man ohne die Beschleunigungsphasen aber nicht aus. Und diese sind ja wohl kaum symmetrisch.

Es sei denn, man würde behaupten, dass Beschleunigungen relativ seien. Das ist aber definitiv nicht so!!!


Alle Zeitangaben in WEZ +1. Es ist jetzt 18:20 Uhr.

Powered by vBulletin® Version 3.8.8 (Deutsch)
Copyright ©2000 - 2024, vBulletin Solutions, Inc.
ScienceUp - Dr. Günter Sturm